LSAT and Law School Admissions Forum

Get expert LSAT preparation and law school admissions advice from PowerScore Test Preparation.

 GLMDYP
  • Posts: 100
  • Joined: Aug 19, 2013
|
#10419
Hi Powerscore!
I simply don't understand what the stimulus is talking about. Can you please explain why (E) is the appropriate choice?
Thanks!
 David Boyle
PowerScore Staff
  • PowerScore Staff
  • Posts: 836
  • Joined: Jun 07, 2013
|
#10472
GLMDYP wrote:Hi Powerscore!
I simply don't understand what the stimulus is talking about. Can you please explain why (E) is the appropriate choice?
Thanks!
Hello,

The stimulus is saying that if a scandal is blamed on a particular political party (rather than on all the parties, say), voters tend to punish that party. (Otherwise, incumbents tend to do pretty well.) (E) follows that logic.

David
 po1016
  • Posts: 4
  • Joined: Nov 25, 2013
|
#13106
For 47, how are A and E diagrammed?
 Ron Gore
PowerScore Staff
  • PowerScore Staff
  • Posts: 220
  • Joined: May 15, 2013
|
#13113
Hi, Po!

In this question, if you were to diagram out the answer choices, you could diagram (A) and (E) like I did below. However, I would consider whether diagramming these answer choices is truly efficient or even necessary.

The stimulus is focused on two scenarios: 1) all parties are blamed equally; and, (2) only one party is blamed. Nothing in the stimulus focuses on what occurs when an incumbent candidate, rather than the party, is responsible for a scandal. So answer choice (A), which presents the responsibility of individual incumbent candidates, rather than parties, as the compound sufficient condition, is totally unconnected to the stimulus. Be cautious about diagramming each answer choice without narrowing down the possibilities first. If you have any thought of completing the section within the allotted, diagramming answer choices should be the exception, rather than the rule.

That being said, the diagrams you requested are:

..... Sufficient ..... :arrow: ..... ..... Necessary

(A):

..... Incumbent(a) responsible for MPS1

..... AND ..... ..... :arrow: ..... consequences same for both Incumbents(a&b)

..... Incumbent(b) responsible for MPS2

(E):
..... MPS resp of party more than individual :arrow: party penalized when possible

Thanks!

Ron
 Jkjones3789
  • Posts: 89
  • Joined: Mar 12, 2014
|
#14515
If the voter's responses are guided by a principle, which one of the following principles would best account for the contrast in reactions described above.

What kind of question stem is this ? I thought it was Evaluate the Argument - PR but I/m not sure.
User avatar
 Dave Killoran
PowerScore Staff
  • PowerScore Staff
  • Posts: 5852
  • Joined: Mar 25, 2011
|
#14519
Hi JK,

This is a Resolve-PR question. "...best account for..." is the same as explain, and since you are trying to explain the "contrast in reactions," you have a Resolve question. Then, when you add in the "principle" element mentioned int he question stem, you get Resolve-PR.

Please let me know if that helps. Thanks!
 taylorballou
  • Posts: 18
  • Joined: Feb 18, 2017
|
#39279
Hello,

Initially I was going to choose E, but then second guessed myself and chose D. I think the last sentence in answer choice E confused me because I didn't think the stimulus showed that degree of certainty (e.g. the party responsible MUST be penalized). After that I think I started getting even more confused when I actually chose D. I believe I made the error of assuming the incumbents (the individuals who make up the parties) were had the same characteristics as the parties themselves. I then interpreted the second half of answer choice D to mean that the chance an incumbent would return to office depended upon whether their opponent was in their same party or belonged to a party responsible for a scandal.

Although I think I know where I went wrong, could you please confirm why answer choice D was wrong?

Thanks,

Taylor
 Eric Ockert
PowerScore Staff
  • PowerScore Staff
  • Posts: 164
  • Joined: Sep 28, 2011
|
#39413
Hi Taylor

Remember, as this is a Resolve the Paradox problem, your goal is to find an answer that, if true, might explain the situation in the stimulus. So here, we are looking for a potential explanation why incumbents don't get punished when blame is spread to all parties, but they do get punished when blame is placed solely on their party.

Answer choice (D) does not really explain this situation. If (D) is true, why aren't the incumbents in the first case punished? It doesn't seem to matter who their challenger is, they are "virtually all" being returned to office.

With answer choice (E), we definitely have the voters blaming either all parties in the first instance, or only one party in the second. But they are definitely blaming parties. So (E) is saying that when you blame a party for a scandal, you penalize the party when possible. That would allow for reelecting incumbents in the first case, because you can't really penalize any one party if they are all to blame. It would also support penalizing the incumbents from the offending party in the second case, because here it is possible to penalize the party.

Finally, the "must" language does not affect answer (E) as this is a Resolve the Paradox question, which falls into the Help Family of Logical Reasoning questions. All we need to find is an answer that could explain the voters' behavior in both situations. Strong wording doesn't really work against the answer, because we don't need to prove anything stated in that answer choice, just determine whether it helps explain the contrast in reactions.

Hope that helps!

Get the most out of your LSAT Prep Plus subscription.

Analyze and track your performance with our Testing and Analytics Package.